IMO 2012 #3
by Wolstenholme, Oct 29, 2014, 2:29 AM
The liar's guessing game is a game played between two players
and
. The rules of the game depend on two positive integers
and
which are known to both players.
At the start of the game
chooses integers
and
with
Player
keeps
secret, and truthfully tells
to player
. Player
now tries to obtain information about
by asking player
questions as follows: each question consists of
specifying an arbitrary set
of positive integers (possibly one specified in some previous question), and asking
whether
belongs to
. Player
may ask as many questions as he wishes. After each question, player
must immediately answer it with yes or no, but is allowed to lie as many times as she wants; the only restriction is that, among any
consecutive answers, at least one answer must be truthful.
After
has asked as many questions as he wants, he must specify a set
of at most
positive integers. If
belongs to
, then
wins; otherwise, he loses. Prove that:
1. If
then
can guarantee a win.
2. For all sufficiently large
, there exists an integer
such that
cannot guarantee a win.
Proof:
So I could only do part
, but I'm assuming this would be worth
points at the IMO?
When I first saw part
, given that asking yes and no questions corresponds to binary digits and the appearance of
, I immediately saw that converting numbers into binary would be important. Given this motivation, let's proceed with the proof!
For the rest of the proof, given a question
(where
is a set) let
if the answer to
is yes and let
otherwise. Also, a number with a line on top of it will be the notation for a string of binary digits. Assume
(if not, the problem is trivial). Consider any
-element subset of
Call this subset
Identify each element of
with a different binary string of length
Let
for all integers
denote the set of numbers in
with a
in the
th position (from left to right) of their binary representation. Now,
ask questions
. After
answers, let
for all integers
.
Now have
ask questions
.
For now, assume that
answers that
for some
and that
is the minimal such number. Then if in reality
we would have that
were all lies. But this is a sequence of
consecutive lies, which is impossible. Therefore we would have that
. We can continue using this method to cancel possibilities for
until we are left with a set of
candidates, as desired.
On the other hand, assume that
answers that
for all integers
. Then one of these must be the truth so we have a set of
candidates, as desired.
Therefore
wins!




At the start of the game



















After






1. If


2. For all sufficiently large



Proof:
So I could only do part


When I first saw part


For the rest of the proof, given a question


![$ [R] = 1 $](http://latex.artofproblemsolving.com/c/b/a/cba8bad4ce0521f737037661dfd8639862d5c70c.png)

![$ [R] = 0 $](http://latex.artofproblemsolving.com/f/d/1/fd1c9acec9733fbc59041f612b8594d5714eeb21.png)














![$ a_i = \overline{[S_1][S_2]\dots[S_i](1 - [S_{i + 1}])(1 - [S_{i + 2}])\dots(1 - [S_{k}])} $](http://latex.artofproblemsolving.com/d/a/3/da3ef61f26d2aad2dc153b465c4b263ae050fa8e.png)

Now have


For now, assume that










On the other hand, assume that




Therefore

This post has been edited 1 time. Last edited by Wolstenholme, Oct 29, 2014, 2:29 AM